A Mathematical Discussion: Fill in the BlankHow do I tell the children's ages?Generalization of Sum and Product puzzleThe heritage of a numismaticGreen Eyed Oracle Variant - Not a Multiple of $17$Find two numbers based on either their product or their sum?Who shall come to the tea party?Another sum and product puzzleAge of the DaughtersAges of mathematician's five childrenRearrange the Sentences, Cross out the Wrong One, Fill in the Blank, and… Do Some Logic!

Did Darth Vader wear the same suit for 20+ years?

Java guess the number

How to make a setting relevant?

Avoiding cliches when writing gods

Traffic law UK, pedestrians

How to skip replacing first occurrence of a character in each line?

How to generate random points without duplication?

You've spoiled/damaged the card

Completing the square to find if quadratic form is positive definite.

PC video game involving floating islands doing aerial combat

Finding x,y coordinates where y is largest

What happened to all the nuclear material being smuggled after the fall of the USSR?

Secure offsite backup, even in the case of hacker root access

Do manufacturers try make their components as close to ideal ones as possible?

Company did not petition for visa in a timely manner. Is asking me to work from overseas, but wants me to take a paycut

What can plausibly explain many of my very long and low-tech bridges?

Their answer is discrete, mine is continuous. They baited me into the wrong answer. I have a P Exam question

How could a government be implemented in a virtual reality?

Pay as you go Or Oyster card

Word for a small burst of laughter that can't be held back

What's the correct term for a waitress in the Middle Ages?

How is it possible that Gollum speaks Westron?

Function to extract float from different price patterns

What happens if you do emergency landing on a US base in middle of the ocean?



A Mathematical Discussion: Fill in the Blank


How do I tell the children's ages?Generalization of Sum and Product puzzleThe heritage of a numismaticGreen Eyed Oracle Variant - Not a Multiple of $17$Find two numbers based on either their product or their sum?Who shall come to the tea party?Another sum and product puzzleAge of the DaughtersAges of mathematician's five childrenRearrange the Sentences, Cross out the Wrong One, Fill in the Blank, and… Do Some Logic!













9












$begingroup$


I originally saw this on r/puzzles, and thought it deserved a place here. I have a solution to this puzzle, but it's really long and ugly. If anyone comes up with something simple, please share!




Two perfect logicians have the following conversation. Fill in the blank with the correct number.



Ana: "Did you know that your favorite number is the sum of the ages of my stuffed animal turtles, and that my favorite number is their product?"



Beth: "I wouldn't know because I don't know your favorite number. If you tell me your favorite number and how many stuffed animal turtles you have, would I know the ages of your stuffed animal turtles?"



Ana: "No."



Beth: "Oh, so your favorite number is ________!"










share|improve this question









$endgroup$











  • $begingroup$
    Is 0 a possible value for age, or are those all strictly positive?
    $endgroup$
    – elias
    May 27 at 9:46










  • $begingroup$
    @elias Strictly positive integers.
    $endgroup$
    – greenturtle3141
    May 27 at 16:14















9












$begingroup$


I originally saw this on r/puzzles, and thought it deserved a place here. I have a solution to this puzzle, but it's really long and ugly. If anyone comes up with something simple, please share!




Two perfect logicians have the following conversation. Fill in the blank with the correct number.



Ana: "Did you know that your favorite number is the sum of the ages of my stuffed animal turtles, and that my favorite number is their product?"



Beth: "I wouldn't know because I don't know your favorite number. If you tell me your favorite number and how many stuffed animal turtles you have, would I know the ages of your stuffed animal turtles?"



Ana: "No."



Beth: "Oh, so your favorite number is ________!"










share|improve this question









$endgroup$











  • $begingroup$
    Is 0 a possible value for age, or are those all strictly positive?
    $endgroup$
    – elias
    May 27 at 9:46










  • $begingroup$
    @elias Strictly positive integers.
    $endgroup$
    – greenturtle3141
    May 27 at 16:14













9












9








9


3



$begingroup$


I originally saw this on r/puzzles, and thought it deserved a place here. I have a solution to this puzzle, but it's really long and ugly. If anyone comes up with something simple, please share!




Two perfect logicians have the following conversation. Fill in the blank with the correct number.



Ana: "Did you know that your favorite number is the sum of the ages of my stuffed animal turtles, and that my favorite number is their product?"



Beth: "I wouldn't know because I don't know your favorite number. If you tell me your favorite number and how many stuffed animal turtles you have, would I know the ages of your stuffed animal turtles?"



Ana: "No."



Beth: "Oh, so your favorite number is ________!"










share|improve this question









$endgroup$




I originally saw this on r/puzzles, and thought it deserved a place here. I have a solution to this puzzle, but it's really long and ugly. If anyone comes up with something simple, please share!




Two perfect logicians have the following conversation. Fill in the blank with the correct number.



Ana: "Did you know that your favorite number is the sum of the ages of my stuffed animal turtles, and that my favorite number is their product?"



Beth: "I wouldn't know because I don't know your favorite number. If you tell me your favorite number and how many stuffed animal turtles you have, would I know the ages of your stuffed animal turtles?"



Ana: "No."



Beth: "Oh, so your favorite number is ________!"







mathematics logical-deduction






share|improve this question













share|improve this question











share|improve this question




share|improve this question










asked May 27 at 8:03









greenturtle3141greenturtle3141

6,18612256




6,18612256











  • $begingroup$
    Is 0 a possible value for age, or are those all strictly positive?
    $endgroup$
    – elias
    May 27 at 9:46










  • $begingroup$
    @elias Strictly positive integers.
    $endgroup$
    – greenturtle3141
    May 27 at 16:14
















  • $begingroup$
    Is 0 a possible value for age, or are those all strictly positive?
    $endgroup$
    – elias
    May 27 at 9:46










  • $begingroup$
    @elias Strictly positive integers.
    $endgroup$
    – greenturtle3141
    May 27 at 16:14















$begingroup$
Is 0 a possible value for age, or are those all strictly positive?
$endgroup$
– elias
May 27 at 9:46




$begingroup$
Is 0 a possible value for age, or are those all strictly positive?
$endgroup$
– elias
May 27 at 9:46












$begingroup$
@elias Strictly positive integers.
$endgroup$
– greenturtle3141
May 27 at 16:14




$begingroup$
@elias Strictly positive integers.
$endgroup$
– greenturtle3141
May 27 at 16:14










4 Answers
4






active

oldest

votes


















7












$begingroup$

We will assume that all the ages of the stuffed animals are natural numbers (i.e. cannot be 0). Removing this assumption makes the problem impossible.



Let $T=...$ be the set of ages of the stuffed animals, $P$ be their product, and $S$ be their sum.



Initially, Beth knows $S$, but not $P$. For some $S$, we need to find a value of $P$ and $|T|$ such that knowing these things does not uniquely identify the members of $T$. Moreover, we know that such a combination is unique for $S$; if it were not unique, then Beth would not be able to determine $P$ simply by knowing that she couldn't determine the values of $T$ knowing $P$ and $|T|$.



First Solution



Lets try some values for $S$. It turns out we need to go as high as




$S=12$




before we find a case of two sets of numbers with the same number of elements and the same product. These are;




$$T=1,3,4,4 implies P=48$$
$$T=2,2,2,6 implies P=48$$




Notice that they have the same sum, product, and number of elements. If Beth knows the product, sum, and number of elements, she still will not know the values.



However, does Beth knowing this help? Only if all other sets of numbers for the sum are uniquely defined. Well, here they are;




- $T=1,1,10 implies P=10$

- $T=1,2,9 implies P=18$

- $T=1,3,8 implies P=24$

- $T=1,4,7 implies P=28$

- $T=1,5,6 implies P=30$

- $T=2,2,8 implies P=32$

- $T=2,3,7 implies P=42$

- $T=2,4,6 implies P=48$

- $T=2,5,5 implies P=50$

- $T=3,3,6 implies P=54$

- $T=3,4,5 implies P=60$

- $T=4,4,4 implies P=64$

- $T=1,1,1,9 implies P=9$

- $T=1,1,2,8 implies P=16$

- $T=1,1,3,7 implies P=21$

- $T=1,1,4,6 implies P=24$

- $T=1,1,5,5 implies P=25$

- $T=1,2,2,7 implies P=28$

- $T=1,2,3,6 implies P=36$

- $T=1,2,4,5 implies P=40$

- $T=1,3,3,5 implies P=45$

- $T=1,3,4,4 implies P=48$ SAME!!

- $T=2,2,2,6 implies P=48$ SAME!!

- $T=2,2,3,5 implies P=60$

- $T=2,2,4,4 implies P=64$

- $T=2,3,3,4 implies P=72$

- $T=3,3,3,3 implies P=81$

- $T=1,1,1,1,8 implies P=8$

- $T=1,1,1,2,7 implies P=14$

- $T=1,1,1,3,6 implies P=18$

- $T=1,1,1,4,5 implies P=20$

- $T=1,1,2,2,6 implies P=24$

- $T=1,1,2,3,5 implies P=30$

- $T=1,1,2,4,4 implies P=32$

- $T=1,2,2,2,5 implies P=40$

- $T=1,2,2,3,4 implies P=48$

- $T=2,2,2,2,4 implies P=64$

- $T=2,2,2,3,3 implies P=72$

- $T=1,1,1,1,1,7 implies P=7$

- $T=1,1,1,1,2,6 implies P=12$

- $T=1,1,1,1,3,5 implies P=15$

- $T=1,1,1,1,4,4 implies P=16$

- $T=1,1,1,2,2,5 implies P=20$

- $T=1,1,1,2,3,4 implies P=24$

- $T=1,1,2,2,2,4 implies P=32$

- $T=1,1,2,2,3,3 implies P=36$

- $T=1,2,2,2,2,3 implies P=48$

- $T=2,2,2,2,2,2 implies P=64$




So all other sets have unique products given the same number of elements. Thus, one solution to this puzzle is:




$$S=12$$




Knowing that it is impossible to know $T$ given $|T|$ and $P$ lets Beth determine that




$$P=48$$




since all other combinations of $|T|$ and $P$ uniquely identify $T$.



Is this unique?



However, are there answers larger than this? Lets explore!




Assume $S ge 13$




Now consider:




- $T=1,6,6,S-13 implies P=36 times (S-13)$

- $T=2,2,9,S-13 implies P=36 times (S-13)$

- $T=1,3,4,4,S-12 implies P=48 times (S-12)$

- $T=2,2,2,6,S-12 implies P=48 times (S-12)$




If $T$ is one of the above and Beth is given $S$, $P$ and $|T|$, then there is no way to uniquely identify the set $T$. Likewise, since there are multiple values for $P$, Beth cannot logically determine the true value of $P$ even if she knows she can't get $T$ from knowing $|T|$ and $P$.



Thus we can rule out




$S ge 13$




Making,




$$S=12, P=48$$




a unique solution.






share|improve this answer











$endgroup$












  • $begingroup$
    This should be the correct answer, you just need to show that nothing greater than 12 for the sum can't work. In addition I'm paranoid, so I'll wait for consensus from everyone else.
    $endgroup$
    – greenturtle3141
    May 27 at 16:27










  • $begingroup$
    I have ruled out $S=13$ since there are multiple possibilities.
    $endgroup$
    – Trenin
    May 27 at 17:49










  • $begingroup$
    So I originally solved this by starting from $S=1$ and bashing through every possibility, working my way up. I guess there doesn't seem to be an easy, intuitive way to obtain 12, unless anyone else has any better ideas.
    $endgroup$
    – greenturtle3141
    May 27 at 22:14










  • $begingroup$
    @greenturtle3141 To be rigorous, I think you need to go through them all. I don't see any mathematical property that can be exploited to find a set of numbers with the same sum and product.
    $endgroup$
    – Trenin
    May 28 at 11:58


















4












$begingroup$

Edit: Trenin has found a smaller value for $X$ which works and I think is correct. Here is the reasoning as to why.



Ana's favourite number is




$48$




Reasoning




As athin pointed out, we are trying to find $X$, $Y$, and $k$ such that there are at least $2$ possibilities of $T = T_1, T_2, dots, T_k$ that satisfies:

$(1)~T_1 + T_2 + dots + T_k = X$
$(2)~T_1 times T_2 times dots times T_k = Y$



Additionally, for the given value of $X$, there must not be a second $Y$ and $k$ for which there are also at least $2$ possibilities of the above occurring. Otherwise, Beth cannot determine the product at the end.


Suppose that Beth's number $X > 14$.
Then, we find that the following two sets of values for $T_i$ have the same sum and product $(2,2,9,X-13)$ and $(1,6,6,X-13)$. Additionally, we have two more sets which also have the same sum and product $(2,2,10, X-14)$ and $(1,5,8,X-14)$. So, in general, Beth cannot determine the product uniquely from the information she's been given.

In the case $X=14$ we have the pair of sets $(1,2,2,9)$ and $(1,1,6,6)$ with product $36$ and the pair of sets $(1,5,8)$ and $(2,2,10)$ with product $40$. Hence, in this case, Beth would not be able to determine whether the product is $36$ or $40$. If $X=13$, we have the pair of sets $(1,1,3,4,4)$ and $(1,2,2,2,6)$ with product of $48$ and the pair of sets $(1,6,6)$ and $(2,9,9)$ with product $36$ so Beth would not be able to determine whether the product is $36$ or $48$ in this case. Overall this implies $X < 13$.

As far as I'm aware, the solution given by Trenin is the only one for the system above with $X < 13$.




Rogue edge case




There is an edge case in the scenario where $X>14$. Namely, if $X=23$, then both products are $360$. But in this case, we can construct another pair of solutions $(2,2,5,5,9)$ and $(1,5,5,6,6)$ whose product is $900$.







share|improve this answer











$endgroup$












  • $begingroup$
    I knew it wasn’t unique to the classic answer, I just was too lazy to find a counter example. Great find!
    $endgroup$
    – El-Guest
    May 27 at 13:25










  • $begingroup$
    But what if Beth's number is 12 and Ana's is 48?
    $endgroup$
    – Trenin
    May 27 at 16:19










  • $begingroup$
    @Trenin You're right, I hadn't spotted this smaller solution.
    $endgroup$
    – hexomino
    May 27 at 16:56


















2












$begingroup$

Partial Solution



Beth's favorite number is:




Not $1$ or $2$, because then Ana's favorite number is trivial to be $1$. This is because of $1 = 1$ and $2 = 1 + 1$. (In case there can be only $1$ turtle, then $2$ is still possible.)


Thus, Beth's favorite number is $X$ where $X geq 3$. This is because of $X$ can be represented as at least $1 + 1 + dots + 1$ and $1 + (X-1)$ so there are at least $2$ possibilities of Ana's favorite number (which is $1$ or $X-1$).




Because Beth still won't know the ages of stuffed animal turtles, given Ana's favorite number and how many stuffed animal turtles, then we can conclude that:




Let's say there are $k$ turtles. If the turtles' ages are $T = T_1, T_2, dots, T_k$; there should be at least $2$ possibilities of $T$ where both sum and product of them are same.




Also as @hexomino pointed out from comments, Beth then know the answer, means:




For such sum, the product should be unique.




To sums up, we want to find:




$X$, $Y$, and $k$ such that there are at least $2$ possibilities of $T = T_1, T_2, dots, T_k$ that satisfies:

$(1)~X geq 3$
$(2)~T_1 + T_2 + dots + T_k = X$
$(3)~T_1 times T_2 times dots times T_k = Y$
$(4)$ There is no other $Y'$ and $k'$ for such $X$ so that $(2), (3)$ are also satisfied for given $Y'$ and $k'$.







share|improve this answer











$endgroup$








  • 1




    $begingroup$
    I think there is an additional restriction in that once you know $X$, you can determine $Y$ because Beth knows exactly the product at the end. This means that there is an $(X,Y,k)$ with at least two solutions and any other such triplet including $X$ has the same value for $Y$. Also we must have the $k >2$, otherwise Beth could determine the turtle values exactly from knowing $Y$ and $k$.
    $endgroup$
    – hexomino
    May 27 at 11:44










  • $begingroup$
    @hexomino ah, clever! I should try to fix this..
    $endgroup$
    – athin
    May 27 at 11:48










  • $begingroup$
    @hexomino wait no, even if you know $X$, $Y$ still couldn't be determined as indicated by first spoiler..
    $endgroup$
    – athin
    May 27 at 11:52










  • $begingroup$
    so Beth's first question is essentially "If you tell me $Y$ and $k$ can I determine the ages?" If $k<3$, then Ana would say "yes" because it's a determined system.
    $endgroup$
    – hexomino
    May 27 at 11:56






  • 1




    $begingroup$
    As to my first point, take your solution $(X,Y,k) = (31, 720, 3)$. For this to work, you have to know that there is no other solution $(31, Y, k)$ which has two different sums with $Y neq 720$, otherwise Beth cannot determine $Y$. In this instance, there is another solution $(1,5,7,18)$ and $(2,3,5,21)$ which has sum $31$, product $630$ $k=4$.
    $endgroup$
    – hexomino
    May 27 at 12:04


















1












$begingroup$


Beth's favourite number can be any number >= 13. The possible age combinations are 6,6,1,[1,1,1,...] and 9,2,2,[1,1,1,...]. You can add as many 1's as you like, because this doesn't change Ana's favourite number. Ana's favourite number must be 36. If it were not, the answer to Beth's question would not give any further information and she would not have asked.




Meanwhile, hexomino gave a good explanation why this is so.






share|improve this answer











$endgroup$












  • $begingroup$
    The problem here is precisely the fact that Beth deduced the number. Consider the four possibilities for $S=13$: 11344 12226 ($P = 48$), and 661 922 ($P = 36$). Since Beth only knows her sum, of which all of these four cases fit, but she also knows that the number of turtles and their product would not uniquely determine the set of agres. This is consistent with both $P=36$ or $P=48$, as both cases have two possible sets of ages. Since there are two possible values for $P$, Beth could not have deduced the ages, contradicting her exclamation that she figured out Ana's number, the product.
    $endgroup$
    – greenturtle3141
    May 27 at 16:24











Your Answer








StackExchange.ready(function()
var channelOptions =
tags: "".split(" "),
id: "559"
;
initTagRenderer("".split(" "), "".split(" "), channelOptions);

StackExchange.using("externalEditor", function()
// Have to fire editor after snippets, if snippets enabled
if (StackExchange.settings.snippets.snippetsEnabled)
StackExchange.using("snippets", function()
createEditor();
);

else
createEditor();

);

function createEditor()
StackExchange.prepareEditor(
heartbeatType: 'answer',
autoActivateHeartbeat: false,
convertImagesToLinks: false,
noModals: true,
showLowRepImageUploadWarning: true,
reputationToPostImages: null,
bindNavPrevention: true,
postfix: "",
imageUploader:
brandingHtml: "Powered by u003ca class="icon-imgur-white" href="https://imgur.com/"u003eu003c/au003e",
contentPolicyHtml: "User contributions licensed under u003ca href="https://creativecommons.org/licenses/by-sa/3.0/"u003ecc by-sa 3.0 with attribution requiredu003c/au003e u003ca href="https://stackoverflow.com/legal/content-policy"u003e(content policy)u003c/au003e",
allowUrls: true
,
noCode: true, onDemand: true,
discardSelector: ".discard-answer"
,immediatelyShowMarkdownHelp:true
);



);













draft saved

draft discarded


















StackExchange.ready(
function ()
StackExchange.openid.initPostLogin('.new-post-login', 'https%3a%2f%2fpuzzling.stackexchange.com%2fquestions%2f84401%2fa-mathematical-discussion-fill-in-the-blank%23new-answer', 'question_page');

);

Post as a guest















Required, but never shown

























4 Answers
4






active

oldest

votes








4 Answers
4






active

oldest

votes









active

oldest

votes






active

oldest

votes









7












$begingroup$

We will assume that all the ages of the stuffed animals are natural numbers (i.e. cannot be 0). Removing this assumption makes the problem impossible.



Let $T=...$ be the set of ages of the stuffed animals, $P$ be their product, and $S$ be their sum.



Initially, Beth knows $S$, but not $P$. For some $S$, we need to find a value of $P$ and $|T|$ such that knowing these things does not uniquely identify the members of $T$. Moreover, we know that such a combination is unique for $S$; if it were not unique, then Beth would not be able to determine $P$ simply by knowing that she couldn't determine the values of $T$ knowing $P$ and $|T|$.



First Solution



Lets try some values for $S$. It turns out we need to go as high as




$S=12$




before we find a case of two sets of numbers with the same number of elements and the same product. These are;




$$T=1,3,4,4 implies P=48$$
$$T=2,2,2,6 implies P=48$$




Notice that they have the same sum, product, and number of elements. If Beth knows the product, sum, and number of elements, she still will not know the values.



However, does Beth knowing this help? Only if all other sets of numbers for the sum are uniquely defined. Well, here they are;




- $T=1,1,10 implies P=10$

- $T=1,2,9 implies P=18$

- $T=1,3,8 implies P=24$

- $T=1,4,7 implies P=28$

- $T=1,5,6 implies P=30$

- $T=2,2,8 implies P=32$

- $T=2,3,7 implies P=42$

- $T=2,4,6 implies P=48$

- $T=2,5,5 implies P=50$

- $T=3,3,6 implies P=54$

- $T=3,4,5 implies P=60$

- $T=4,4,4 implies P=64$

- $T=1,1,1,9 implies P=9$

- $T=1,1,2,8 implies P=16$

- $T=1,1,3,7 implies P=21$

- $T=1,1,4,6 implies P=24$

- $T=1,1,5,5 implies P=25$

- $T=1,2,2,7 implies P=28$

- $T=1,2,3,6 implies P=36$

- $T=1,2,4,5 implies P=40$

- $T=1,3,3,5 implies P=45$

- $T=1,3,4,4 implies P=48$ SAME!!

- $T=2,2,2,6 implies P=48$ SAME!!

- $T=2,2,3,5 implies P=60$

- $T=2,2,4,4 implies P=64$

- $T=2,3,3,4 implies P=72$

- $T=3,3,3,3 implies P=81$

- $T=1,1,1,1,8 implies P=8$

- $T=1,1,1,2,7 implies P=14$

- $T=1,1,1,3,6 implies P=18$

- $T=1,1,1,4,5 implies P=20$

- $T=1,1,2,2,6 implies P=24$

- $T=1,1,2,3,5 implies P=30$

- $T=1,1,2,4,4 implies P=32$

- $T=1,2,2,2,5 implies P=40$

- $T=1,2,2,3,4 implies P=48$

- $T=2,2,2,2,4 implies P=64$

- $T=2,2,2,3,3 implies P=72$

- $T=1,1,1,1,1,7 implies P=7$

- $T=1,1,1,1,2,6 implies P=12$

- $T=1,1,1,1,3,5 implies P=15$

- $T=1,1,1,1,4,4 implies P=16$

- $T=1,1,1,2,2,5 implies P=20$

- $T=1,1,1,2,3,4 implies P=24$

- $T=1,1,2,2,2,4 implies P=32$

- $T=1,1,2,2,3,3 implies P=36$

- $T=1,2,2,2,2,3 implies P=48$

- $T=2,2,2,2,2,2 implies P=64$




So all other sets have unique products given the same number of elements. Thus, one solution to this puzzle is:




$$S=12$$




Knowing that it is impossible to know $T$ given $|T|$ and $P$ lets Beth determine that




$$P=48$$




since all other combinations of $|T|$ and $P$ uniquely identify $T$.



Is this unique?



However, are there answers larger than this? Lets explore!




Assume $S ge 13$




Now consider:




- $T=1,6,6,S-13 implies P=36 times (S-13)$

- $T=2,2,9,S-13 implies P=36 times (S-13)$

- $T=1,3,4,4,S-12 implies P=48 times (S-12)$

- $T=2,2,2,6,S-12 implies P=48 times (S-12)$




If $T$ is one of the above and Beth is given $S$, $P$ and $|T|$, then there is no way to uniquely identify the set $T$. Likewise, since there are multiple values for $P$, Beth cannot logically determine the true value of $P$ even if she knows she can't get $T$ from knowing $|T|$ and $P$.



Thus we can rule out




$S ge 13$




Making,




$$S=12, P=48$$




a unique solution.






share|improve this answer











$endgroup$












  • $begingroup$
    This should be the correct answer, you just need to show that nothing greater than 12 for the sum can't work. In addition I'm paranoid, so I'll wait for consensus from everyone else.
    $endgroup$
    – greenturtle3141
    May 27 at 16:27










  • $begingroup$
    I have ruled out $S=13$ since there are multiple possibilities.
    $endgroup$
    – Trenin
    May 27 at 17:49










  • $begingroup$
    So I originally solved this by starting from $S=1$ and bashing through every possibility, working my way up. I guess there doesn't seem to be an easy, intuitive way to obtain 12, unless anyone else has any better ideas.
    $endgroup$
    – greenturtle3141
    May 27 at 22:14










  • $begingroup$
    @greenturtle3141 To be rigorous, I think you need to go through them all. I don't see any mathematical property that can be exploited to find a set of numbers with the same sum and product.
    $endgroup$
    – Trenin
    May 28 at 11:58















7












$begingroup$

We will assume that all the ages of the stuffed animals are natural numbers (i.e. cannot be 0). Removing this assumption makes the problem impossible.



Let $T=...$ be the set of ages of the stuffed animals, $P$ be their product, and $S$ be their sum.



Initially, Beth knows $S$, but not $P$. For some $S$, we need to find a value of $P$ and $|T|$ such that knowing these things does not uniquely identify the members of $T$. Moreover, we know that such a combination is unique for $S$; if it were not unique, then Beth would not be able to determine $P$ simply by knowing that she couldn't determine the values of $T$ knowing $P$ and $|T|$.



First Solution



Lets try some values for $S$. It turns out we need to go as high as




$S=12$




before we find a case of two sets of numbers with the same number of elements and the same product. These are;




$$T=1,3,4,4 implies P=48$$
$$T=2,2,2,6 implies P=48$$




Notice that they have the same sum, product, and number of elements. If Beth knows the product, sum, and number of elements, she still will not know the values.



However, does Beth knowing this help? Only if all other sets of numbers for the sum are uniquely defined. Well, here they are;




- $T=1,1,10 implies P=10$

- $T=1,2,9 implies P=18$

- $T=1,3,8 implies P=24$

- $T=1,4,7 implies P=28$

- $T=1,5,6 implies P=30$

- $T=2,2,8 implies P=32$

- $T=2,3,7 implies P=42$

- $T=2,4,6 implies P=48$

- $T=2,5,5 implies P=50$

- $T=3,3,6 implies P=54$

- $T=3,4,5 implies P=60$

- $T=4,4,4 implies P=64$

- $T=1,1,1,9 implies P=9$

- $T=1,1,2,8 implies P=16$

- $T=1,1,3,7 implies P=21$

- $T=1,1,4,6 implies P=24$

- $T=1,1,5,5 implies P=25$

- $T=1,2,2,7 implies P=28$

- $T=1,2,3,6 implies P=36$

- $T=1,2,4,5 implies P=40$

- $T=1,3,3,5 implies P=45$

- $T=1,3,4,4 implies P=48$ SAME!!

- $T=2,2,2,6 implies P=48$ SAME!!

- $T=2,2,3,5 implies P=60$

- $T=2,2,4,4 implies P=64$

- $T=2,3,3,4 implies P=72$

- $T=3,3,3,3 implies P=81$

- $T=1,1,1,1,8 implies P=8$

- $T=1,1,1,2,7 implies P=14$

- $T=1,1,1,3,6 implies P=18$

- $T=1,1,1,4,5 implies P=20$

- $T=1,1,2,2,6 implies P=24$

- $T=1,1,2,3,5 implies P=30$

- $T=1,1,2,4,4 implies P=32$

- $T=1,2,2,2,5 implies P=40$

- $T=1,2,2,3,4 implies P=48$

- $T=2,2,2,2,4 implies P=64$

- $T=2,2,2,3,3 implies P=72$

- $T=1,1,1,1,1,7 implies P=7$

- $T=1,1,1,1,2,6 implies P=12$

- $T=1,1,1,1,3,5 implies P=15$

- $T=1,1,1,1,4,4 implies P=16$

- $T=1,1,1,2,2,5 implies P=20$

- $T=1,1,1,2,3,4 implies P=24$

- $T=1,1,2,2,2,4 implies P=32$

- $T=1,1,2,2,3,3 implies P=36$

- $T=1,2,2,2,2,3 implies P=48$

- $T=2,2,2,2,2,2 implies P=64$




So all other sets have unique products given the same number of elements. Thus, one solution to this puzzle is:




$$S=12$$




Knowing that it is impossible to know $T$ given $|T|$ and $P$ lets Beth determine that




$$P=48$$




since all other combinations of $|T|$ and $P$ uniquely identify $T$.



Is this unique?



However, are there answers larger than this? Lets explore!




Assume $S ge 13$




Now consider:




- $T=1,6,6,S-13 implies P=36 times (S-13)$

- $T=2,2,9,S-13 implies P=36 times (S-13)$

- $T=1,3,4,4,S-12 implies P=48 times (S-12)$

- $T=2,2,2,6,S-12 implies P=48 times (S-12)$




If $T$ is one of the above and Beth is given $S$, $P$ and $|T|$, then there is no way to uniquely identify the set $T$. Likewise, since there are multiple values for $P$, Beth cannot logically determine the true value of $P$ even if she knows she can't get $T$ from knowing $|T|$ and $P$.



Thus we can rule out




$S ge 13$




Making,




$$S=12, P=48$$




a unique solution.






share|improve this answer











$endgroup$












  • $begingroup$
    This should be the correct answer, you just need to show that nothing greater than 12 for the sum can't work. In addition I'm paranoid, so I'll wait for consensus from everyone else.
    $endgroup$
    – greenturtle3141
    May 27 at 16:27










  • $begingroup$
    I have ruled out $S=13$ since there are multiple possibilities.
    $endgroup$
    – Trenin
    May 27 at 17:49










  • $begingroup$
    So I originally solved this by starting from $S=1$ and bashing through every possibility, working my way up. I guess there doesn't seem to be an easy, intuitive way to obtain 12, unless anyone else has any better ideas.
    $endgroup$
    – greenturtle3141
    May 27 at 22:14










  • $begingroup$
    @greenturtle3141 To be rigorous, I think you need to go through them all. I don't see any mathematical property that can be exploited to find a set of numbers with the same sum and product.
    $endgroup$
    – Trenin
    May 28 at 11:58













7












7








7





$begingroup$

We will assume that all the ages of the stuffed animals are natural numbers (i.e. cannot be 0). Removing this assumption makes the problem impossible.



Let $T=...$ be the set of ages of the stuffed animals, $P$ be their product, and $S$ be their sum.



Initially, Beth knows $S$, but not $P$. For some $S$, we need to find a value of $P$ and $|T|$ such that knowing these things does not uniquely identify the members of $T$. Moreover, we know that such a combination is unique for $S$; if it were not unique, then Beth would not be able to determine $P$ simply by knowing that she couldn't determine the values of $T$ knowing $P$ and $|T|$.



First Solution



Lets try some values for $S$. It turns out we need to go as high as




$S=12$




before we find a case of two sets of numbers with the same number of elements and the same product. These are;




$$T=1,3,4,4 implies P=48$$
$$T=2,2,2,6 implies P=48$$




Notice that they have the same sum, product, and number of elements. If Beth knows the product, sum, and number of elements, she still will not know the values.



However, does Beth knowing this help? Only if all other sets of numbers for the sum are uniquely defined. Well, here they are;




- $T=1,1,10 implies P=10$

- $T=1,2,9 implies P=18$

- $T=1,3,8 implies P=24$

- $T=1,4,7 implies P=28$

- $T=1,5,6 implies P=30$

- $T=2,2,8 implies P=32$

- $T=2,3,7 implies P=42$

- $T=2,4,6 implies P=48$

- $T=2,5,5 implies P=50$

- $T=3,3,6 implies P=54$

- $T=3,4,5 implies P=60$

- $T=4,4,4 implies P=64$

- $T=1,1,1,9 implies P=9$

- $T=1,1,2,8 implies P=16$

- $T=1,1,3,7 implies P=21$

- $T=1,1,4,6 implies P=24$

- $T=1,1,5,5 implies P=25$

- $T=1,2,2,7 implies P=28$

- $T=1,2,3,6 implies P=36$

- $T=1,2,4,5 implies P=40$

- $T=1,3,3,5 implies P=45$

- $T=1,3,4,4 implies P=48$ SAME!!

- $T=2,2,2,6 implies P=48$ SAME!!

- $T=2,2,3,5 implies P=60$

- $T=2,2,4,4 implies P=64$

- $T=2,3,3,4 implies P=72$

- $T=3,3,3,3 implies P=81$

- $T=1,1,1,1,8 implies P=8$

- $T=1,1,1,2,7 implies P=14$

- $T=1,1,1,3,6 implies P=18$

- $T=1,1,1,4,5 implies P=20$

- $T=1,1,2,2,6 implies P=24$

- $T=1,1,2,3,5 implies P=30$

- $T=1,1,2,4,4 implies P=32$

- $T=1,2,2,2,5 implies P=40$

- $T=1,2,2,3,4 implies P=48$

- $T=2,2,2,2,4 implies P=64$

- $T=2,2,2,3,3 implies P=72$

- $T=1,1,1,1,1,7 implies P=7$

- $T=1,1,1,1,2,6 implies P=12$

- $T=1,1,1,1,3,5 implies P=15$

- $T=1,1,1,1,4,4 implies P=16$

- $T=1,1,1,2,2,5 implies P=20$

- $T=1,1,1,2,3,4 implies P=24$

- $T=1,1,2,2,2,4 implies P=32$

- $T=1,1,2,2,3,3 implies P=36$

- $T=1,2,2,2,2,3 implies P=48$

- $T=2,2,2,2,2,2 implies P=64$




So all other sets have unique products given the same number of elements. Thus, one solution to this puzzle is:




$$S=12$$




Knowing that it is impossible to know $T$ given $|T|$ and $P$ lets Beth determine that




$$P=48$$




since all other combinations of $|T|$ and $P$ uniquely identify $T$.



Is this unique?



However, are there answers larger than this? Lets explore!




Assume $S ge 13$




Now consider:




- $T=1,6,6,S-13 implies P=36 times (S-13)$

- $T=2,2,9,S-13 implies P=36 times (S-13)$

- $T=1,3,4,4,S-12 implies P=48 times (S-12)$

- $T=2,2,2,6,S-12 implies P=48 times (S-12)$




If $T$ is one of the above and Beth is given $S$, $P$ and $|T|$, then there is no way to uniquely identify the set $T$. Likewise, since there are multiple values for $P$, Beth cannot logically determine the true value of $P$ even if she knows she can't get $T$ from knowing $|T|$ and $P$.



Thus we can rule out




$S ge 13$




Making,




$$S=12, P=48$$




a unique solution.






share|improve this answer











$endgroup$



We will assume that all the ages of the stuffed animals are natural numbers (i.e. cannot be 0). Removing this assumption makes the problem impossible.



Let $T=...$ be the set of ages of the stuffed animals, $P$ be their product, and $S$ be their sum.



Initially, Beth knows $S$, but not $P$. For some $S$, we need to find a value of $P$ and $|T|$ such that knowing these things does not uniquely identify the members of $T$. Moreover, we know that such a combination is unique for $S$; if it were not unique, then Beth would not be able to determine $P$ simply by knowing that she couldn't determine the values of $T$ knowing $P$ and $|T|$.



First Solution



Lets try some values for $S$. It turns out we need to go as high as




$S=12$




before we find a case of two sets of numbers with the same number of elements and the same product. These are;




$$T=1,3,4,4 implies P=48$$
$$T=2,2,2,6 implies P=48$$




Notice that they have the same sum, product, and number of elements. If Beth knows the product, sum, and number of elements, she still will not know the values.



However, does Beth knowing this help? Only if all other sets of numbers for the sum are uniquely defined. Well, here they are;




- $T=1,1,10 implies P=10$

- $T=1,2,9 implies P=18$

- $T=1,3,8 implies P=24$

- $T=1,4,7 implies P=28$

- $T=1,5,6 implies P=30$

- $T=2,2,8 implies P=32$

- $T=2,3,7 implies P=42$

- $T=2,4,6 implies P=48$

- $T=2,5,5 implies P=50$

- $T=3,3,6 implies P=54$

- $T=3,4,5 implies P=60$

- $T=4,4,4 implies P=64$

- $T=1,1,1,9 implies P=9$

- $T=1,1,2,8 implies P=16$

- $T=1,1,3,7 implies P=21$

- $T=1,1,4,6 implies P=24$

- $T=1,1,5,5 implies P=25$

- $T=1,2,2,7 implies P=28$

- $T=1,2,3,6 implies P=36$

- $T=1,2,4,5 implies P=40$

- $T=1,3,3,5 implies P=45$

- $T=1,3,4,4 implies P=48$ SAME!!

- $T=2,2,2,6 implies P=48$ SAME!!

- $T=2,2,3,5 implies P=60$

- $T=2,2,4,4 implies P=64$

- $T=2,3,3,4 implies P=72$

- $T=3,3,3,3 implies P=81$

- $T=1,1,1,1,8 implies P=8$

- $T=1,1,1,2,7 implies P=14$

- $T=1,1,1,3,6 implies P=18$

- $T=1,1,1,4,5 implies P=20$

- $T=1,1,2,2,6 implies P=24$

- $T=1,1,2,3,5 implies P=30$

- $T=1,1,2,4,4 implies P=32$

- $T=1,2,2,2,5 implies P=40$

- $T=1,2,2,3,4 implies P=48$

- $T=2,2,2,2,4 implies P=64$

- $T=2,2,2,3,3 implies P=72$

- $T=1,1,1,1,1,7 implies P=7$

- $T=1,1,1,1,2,6 implies P=12$

- $T=1,1,1,1,3,5 implies P=15$

- $T=1,1,1,1,4,4 implies P=16$

- $T=1,1,1,2,2,5 implies P=20$

- $T=1,1,1,2,3,4 implies P=24$

- $T=1,1,2,2,2,4 implies P=32$

- $T=1,1,2,2,3,3 implies P=36$

- $T=1,2,2,2,2,3 implies P=48$

- $T=2,2,2,2,2,2 implies P=64$




So all other sets have unique products given the same number of elements. Thus, one solution to this puzzle is:




$$S=12$$




Knowing that it is impossible to know $T$ given $|T|$ and $P$ lets Beth determine that




$$P=48$$




since all other combinations of $|T|$ and $P$ uniquely identify $T$.



Is this unique?



However, are there answers larger than this? Lets explore!




Assume $S ge 13$




Now consider:




- $T=1,6,6,S-13 implies P=36 times (S-13)$

- $T=2,2,9,S-13 implies P=36 times (S-13)$

- $T=1,3,4,4,S-12 implies P=48 times (S-12)$

- $T=2,2,2,6,S-12 implies P=48 times (S-12)$




If $T$ is one of the above and Beth is given $S$, $P$ and $|T|$, then there is no way to uniquely identify the set $T$. Likewise, since there are multiple values for $P$, Beth cannot logically determine the true value of $P$ even if she knows she can't get $T$ from knowing $|T|$ and $P$.



Thus we can rule out




$S ge 13$




Making,




$$S=12, P=48$$




a unique solution.







share|improve this answer














share|improve this answer



share|improve this answer








edited May 27 at 18:12

























answered May 27 at 16:10









TreninTrenin

8,0231645




8,0231645











  • $begingroup$
    This should be the correct answer, you just need to show that nothing greater than 12 for the sum can't work. In addition I'm paranoid, so I'll wait for consensus from everyone else.
    $endgroup$
    – greenturtle3141
    May 27 at 16:27










  • $begingroup$
    I have ruled out $S=13$ since there are multiple possibilities.
    $endgroup$
    – Trenin
    May 27 at 17:49










  • $begingroup$
    So I originally solved this by starting from $S=1$ and bashing through every possibility, working my way up. I guess there doesn't seem to be an easy, intuitive way to obtain 12, unless anyone else has any better ideas.
    $endgroup$
    – greenturtle3141
    May 27 at 22:14










  • $begingroup$
    @greenturtle3141 To be rigorous, I think you need to go through them all. I don't see any mathematical property that can be exploited to find a set of numbers with the same sum and product.
    $endgroup$
    – Trenin
    May 28 at 11:58
















  • $begingroup$
    This should be the correct answer, you just need to show that nothing greater than 12 for the sum can't work. In addition I'm paranoid, so I'll wait for consensus from everyone else.
    $endgroup$
    – greenturtle3141
    May 27 at 16:27










  • $begingroup$
    I have ruled out $S=13$ since there are multiple possibilities.
    $endgroup$
    – Trenin
    May 27 at 17:49










  • $begingroup$
    So I originally solved this by starting from $S=1$ and bashing through every possibility, working my way up. I guess there doesn't seem to be an easy, intuitive way to obtain 12, unless anyone else has any better ideas.
    $endgroup$
    – greenturtle3141
    May 27 at 22:14










  • $begingroup$
    @greenturtle3141 To be rigorous, I think you need to go through them all. I don't see any mathematical property that can be exploited to find a set of numbers with the same sum and product.
    $endgroup$
    – Trenin
    May 28 at 11:58















$begingroup$
This should be the correct answer, you just need to show that nothing greater than 12 for the sum can't work. In addition I'm paranoid, so I'll wait for consensus from everyone else.
$endgroup$
– greenturtle3141
May 27 at 16:27




$begingroup$
This should be the correct answer, you just need to show that nothing greater than 12 for the sum can't work. In addition I'm paranoid, so I'll wait for consensus from everyone else.
$endgroup$
– greenturtle3141
May 27 at 16:27












$begingroup$
I have ruled out $S=13$ since there are multiple possibilities.
$endgroup$
– Trenin
May 27 at 17:49




$begingroup$
I have ruled out $S=13$ since there are multiple possibilities.
$endgroup$
– Trenin
May 27 at 17:49












$begingroup$
So I originally solved this by starting from $S=1$ and bashing through every possibility, working my way up. I guess there doesn't seem to be an easy, intuitive way to obtain 12, unless anyone else has any better ideas.
$endgroup$
– greenturtle3141
May 27 at 22:14




$begingroup$
So I originally solved this by starting from $S=1$ and bashing through every possibility, working my way up. I guess there doesn't seem to be an easy, intuitive way to obtain 12, unless anyone else has any better ideas.
$endgroup$
– greenturtle3141
May 27 at 22:14












$begingroup$
@greenturtle3141 To be rigorous, I think you need to go through them all. I don't see any mathematical property that can be exploited to find a set of numbers with the same sum and product.
$endgroup$
– Trenin
May 28 at 11:58




$begingroup$
@greenturtle3141 To be rigorous, I think you need to go through them all. I don't see any mathematical property that can be exploited to find a set of numbers with the same sum and product.
$endgroup$
– Trenin
May 28 at 11:58











4












$begingroup$

Edit: Trenin has found a smaller value for $X$ which works and I think is correct. Here is the reasoning as to why.



Ana's favourite number is




$48$




Reasoning




As athin pointed out, we are trying to find $X$, $Y$, and $k$ such that there are at least $2$ possibilities of $T = T_1, T_2, dots, T_k$ that satisfies:

$(1)~T_1 + T_2 + dots + T_k = X$
$(2)~T_1 times T_2 times dots times T_k = Y$



Additionally, for the given value of $X$, there must not be a second $Y$ and $k$ for which there are also at least $2$ possibilities of the above occurring. Otherwise, Beth cannot determine the product at the end.


Suppose that Beth's number $X > 14$.
Then, we find that the following two sets of values for $T_i$ have the same sum and product $(2,2,9,X-13)$ and $(1,6,6,X-13)$. Additionally, we have two more sets which also have the same sum and product $(2,2,10, X-14)$ and $(1,5,8,X-14)$. So, in general, Beth cannot determine the product uniquely from the information she's been given.

In the case $X=14$ we have the pair of sets $(1,2,2,9)$ and $(1,1,6,6)$ with product $36$ and the pair of sets $(1,5,8)$ and $(2,2,10)$ with product $40$. Hence, in this case, Beth would not be able to determine whether the product is $36$ or $40$. If $X=13$, we have the pair of sets $(1,1,3,4,4)$ and $(1,2,2,2,6)$ with product of $48$ and the pair of sets $(1,6,6)$ and $(2,9,9)$ with product $36$ so Beth would not be able to determine whether the product is $36$ or $48$ in this case. Overall this implies $X < 13$.

As far as I'm aware, the solution given by Trenin is the only one for the system above with $X < 13$.




Rogue edge case




There is an edge case in the scenario where $X>14$. Namely, if $X=23$, then both products are $360$. But in this case, we can construct another pair of solutions $(2,2,5,5,9)$ and $(1,5,5,6,6)$ whose product is $900$.







share|improve this answer











$endgroup$












  • $begingroup$
    I knew it wasn’t unique to the classic answer, I just was too lazy to find a counter example. Great find!
    $endgroup$
    – El-Guest
    May 27 at 13:25










  • $begingroup$
    But what if Beth's number is 12 and Ana's is 48?
    $endgroup$
    – Trenin
    May 27 at 16:19










  • $begingroup$
    @Trenin You're right, I hadn't spotted this smaller solution.
    $endgroup$
    – hexomino
    May 27 at 16:56















4












$begingroup$

Edit: Trenin has found a smaller value for $X$ which works and I think is correct. Here is the reasoning as to why.



Ana's favourite number is




$48$




Reasoning




As athin pointed out, we are trying to find $X$, $Y$, and $k$ such that there are at least $2$ possibilities of $T = T_1, T_2, dots, T_k$ that satisfies:

$(1)~T_1 + T_2 + dots + T_k = X$
$(2)~T_1 times T_2 times dots times T_k = Y$



Additionally, for the given value of $X$, there must not be a second $Y$ and $k$ for which there are also at least $2$ possibilities of the above occurring. Otherwise, Beth cannot determine the product at the end.


Suppose that Beth's number $X > 14$.
Then, we find that the following two sets of values for $T_i$ have the same sum and product $(2,2,9,X-13)$ and $(1,6,6,X-13)$. Additionally, we have two more sets which also have the same sum and product $(2,2,10, X-14)$ and $(1,5,8,X-14)$. So, in general, Beth cannot determine the product uniquely from the information she's been given.

In the case $X=14$ we have the pair of sets $(1,2,2,9)$ and $(1,1,6,6)$ with product $36$ and the pair of sets $(1,5,8)$ and $(2,2,10)$ with product $40$. Hence, in this case, Beth would not be able to determine whether the product is $36$ or $40$. If $X=13$, we have the pair of sets $(1,1,3,4,4)$ and $(1,2,2,2,6)$ with product of $48$ and the pair of sets $(1,6,6)$ and $(2,9,9)$ with product $36$ so Beth would not be able to determine whether the product is $36$ or $48$ in this case. Overall this implies $X < 13$.

As far as I'm aware, the solution given by Trenin is the only one for the system above with $X < 13$.




Rogue edge case




There is an edge case in the scenario where $X>14$. Namely, if $X=23$, then both products are $360$. But in this case, we can construct another pair of solutions $(2,2,5,5,9)$ and $(1,5,5,6,6)$ whose product is $900$.







share|improve this answer











$endgroup$












  • $begingroup$
    I knew it wasn’t unique to the classic answer, I just was too lazy to find a counter example. Great find!
    $endgroup$
    – El-Guest
    May 27 at 13:25










  • $begingroup$
    But what if Beth's number is 12 and Ana's is 48?
    $endgroup$
    – Trenin
    May 27 at 16:19










  • $begingroup$
    @Trenin You're right, I hadn't spotted this smaller solution.
    $endgroup$
    – hexomino
    May 27 at 16:56













4












4








4





$begingroup$

Edit: Trenin has found a smaller value for $X$ which works and I think is correct. Here is the reasoning as to why.



Ana's favourite number is




$48$




Reasoning




As athin pointed out, we are trying to find $X$, $Y$, and $k$ such that there are at least $2$ possibilities of $T = T_1, T_2, dots, T_k$ that satisfies:

$(1)~T_1 + T_2 + dots + T_k = X$
$(2)~T_1 times T_2 times dots times T_k = Y$



Additionally, for the given value of $X$, there must not be a second $Y$ and $k$ for which there are also at least $2$ possibilities of the above occurring. Otherwise, Beth cannot determine the product at the end.


Suppose that Beth's number $X > 14$.
Then, we find that the following two sets of values for $T_i$ have the same sum and product $(2,2,9,X-13)$ and $(1,6,6,X-13)$. Additionally, we have two more sets which also have the same sum and product $(2,2,10, X-14)$ and $(1,5,8,X-14)$. So, in general, Beth cannot determine the product uniquely from the information she's been given.

In the case $X=14$ we have the pair of sets $(1,2,2,9)$ and $(1,1,6,6)$ with product $36$ and the pair of sets $(1,5,8)$ and $(2,2,10)$ with product $40$. Hence, in this case, Beth would not be able to determine whether the product is $36$ or $40$. If $X=13$, we have the pair of sets $(1,1,3,4,4)$ and $(1,2,2,2,6)$ with product of $48$ and the pair of sets $(1,6,6)$ and $(2,9,9)$ with product $36$ so Beth would not be able to determine whether the product is $36$ or $48$ in this case. Overall this implies $X < 13$.

As far as I'm aware, the solution given by Trenin is the only one for the system above with $X < 13$.




Rogue edge case




There is an edge case in the scenario where $X>14$. Namely, if $X=23$, then both products are $360$. But in this case, we can construct another pair of solutions $(2,2,5,5,9)$ and $(1,5,5,6,6)$ whose product is $900$.







share|improve this answer











$endgroup$



Edit: Trenin has found a smaller value for $X$ which works and I think is correct. Here is the reasoning as to why.



Ana's favourite number is




$48$




Reasoning




As athin pointed out, we are trying to find $X$, $Y$, and $k$ such that there are at least $2$ possibilities of $T = T_1, T_2, dots, T_k$ that satisfies:

$(1)~T_1 + T_2 + dots + T_k = X$
$(2)~T_1 times T_2 times dots times T_k = Y$



Additionally, for the given value of $X$, there must not be a second $Y$ and $k$ for which there are also at least $2$ possibilities of the above occurring. Otherwise, Beth cannot determine the product at the end.


Suppose that Beth's number $X > 14$.
Then, we find that the following two sets of values for $T_i$ have the same sum and product $(2,2,9,X-13)$ and $(1,6,6,X-13)$. Additionally, we have two more sets which also have the same sum and product $(2,2,10, X-14)$ and $(1,5,8,X-14)$. So, in general, Beth cannot determine the product uniquely from the information she's been given.

In the case $X=14$ we have the pair of sets $(1,2,2,9)$ and $(1,1,6,6)$ with product $36$ and the pair of sets $(1,5,8)$ and $(2,2,10)$ with product $40$. Hence, in this case, Beth would not be able to determine whether the product is $36$ or $40$. If $X=13$, we have the pair of sets $(1,1,3,4,4)$ and $(1,2,2,2,6)$ with product of $48$ and the pair of sets $(1,6,6)$ and $(2,9,9)$ with product $36$ so Beth would not be able to determine whether the product is $36$ or $48$ in this case. Overall this implies $X < 13$.

As far as I'm aware, the solution given by Trenin is the only one for the system above with $X < 13$.




Rogue edge case




There is an edge case in the scenario where $X>14$. Namely, if $X=23$, then both products are $360$. But in this case, we can construct another pair of solutions $(2,2,5,5,9)$ and $(1,5,5,6,6)$ whose product is $900$.








share|improve this answer














share|improve this answer



share|improve this answer








edited May 27 at 17:00

























answered May 27 at 12:17









hexominohexomino

51.7k4152245




51.7k4152245











  • $begingroup$
    I knew it wasn’t unique to the classic answer, I just was too lazy to find a counter example. Great find!
    $endgroup$
    – El-Guest
    May 27 at 13:25










  • $begingroup$
    But what if Beth's number is 12 and Ana's is 48?
    $endgroup$
    – Trenin
    May 27 at 16:19










  • $begingroup$
    @Trenin You're right, I hadn't spotted this smaller solution.
    $endgroup$
    – hexomino
    May 27 at 16:56
















  • $begingroup$
    I knew it wasn’t unique to the classic answer, I just was too lazy to find a counter example. Great find!
    $endgroup$
    – El-Guest
    May 27 at 13:25










  • $begingroup$
    But what if Beth's number is 12 and Ana's is 48?
    $endgroup$
    – Trenin
    May 27 at 16:19










  • $begingroup$
    @Trenin You're right, I hadn't spotted this smaller solution.
    $endgroup$
    – hexomino
    May 27 at 16:56















$begingroup$
I knew it wasn’t unique to the classic answer, I just was too lazy to find a counter example. Great find!
$endgroup$
– El-Guest
May 27 at 13:25




$begingroup$
I knew it wasn’t unique to the classic answer, I just was too lazy to find a counter example. Great find!
$endgroup$
– El-Guest
May 27 at 13:25












$begingroup$
But what if Beth's number is 12 and Ana's is 48?
$endgroup$
– Trenin
May 27 at 16:19




$begingroup$
But what if Beth's number is 12 and Ana's is 48?
$endgroup$
– Trenin
May 27 at 16:19












$begingroup$
@Trenin You're right, I hadn't spotted this smaller solution.
$endgroup$
– hexomino
May 27 at 16:56




$begingroup$
@Trenin You're right, I hadn't spotted this smaller solution.
$endgroup$
– hexomino
May 27 at 16:56











2












$begingroup$

Partial Solution



Beth's favorite number is:




Not $1$ or $2$, because then Ana's favorite number is trivial to be $1$. This is because of $1 = 1$ and $2 = 1 + 1$. (In case there can be only $1$ turtle, then $2$ is still possible.)


Thus, Beth's favorite number is $X$ where $X geq 3$. This is because of $X$ can be represented as at least $1 + 1 + dots + 1$ and $1 + (X-1)$ so there are at least $2$ possibilities of Ana's favorite number (which is $1$ or $X-1$).




Because Beth still won't know the ages of stuffed animal turtles, given Ana's favorite number and how many stuffed animal turtles, then we can conclude that:




Let's say there are $k$ turtles. If the turtles' ages are $T = T_1, T_2, dots, T_k$; there should be at least $2$ possibilities of $T$ where both sum and product of them are same.




Also as @hexomino pointed out from comments, Beth then know the answer, means:




For such sum, the product should be unique.




To sums up, we want to find:




$X$, $Y$, and $k$ such that there are at least $2$ possibilities of $T = T_1, T_2, dots, T_k$ that satisfies:

$(1)~X geq 3$
$(2)~T_1 + T_2 + dots + T_k = X$
$(3)~T_1 times T_2 times dots times T_k = Y$
$(4)$ There is no other $Y'$ and $k'$ for such $X$ so that $(2), (3)$ are also satisfied for given $Y'$ and $k'$.







share|improve this answer











$endgroup$








  • 1




    $begingroup$
    I think there is an additional restriction in that once you know $X$, you can determine $Y$ because Beth knows exactly the product at the end. This means that there is an $(X,Y,k)$ with at least two solutions and any other such triplet including $X$ has the same value for $Y$. Also we must have the $k >2$, otherwise Beth could determine the turtle values exactly from knowing $Y$ and $k$.
    $endgroup$
    – hexomino
    May 27 at 11:44










  • $begingroup$
    @hexomino ah, clever! I should try to fix this..
    $endgroup$
    – athin
    May 27 at 11:48










  • $begingroup$
    @hexomino wait no, even if you know $X$, $Y$ still couldn't be determined as indicated by first spoiler..
    $endgroup$
    – athin
    May 27 at 11:52










  • $begingroup$
    so Beth's first question is essentially "If you tell me $Y$ and $k$ can I determine the ages?" If $k<3$, then Ana would say "yes" because it's a determined system.
    $endgroup$
    – hexomino
    May 27 at 11:56






  • 1




    $begingroup$
    As to my first point, take your solution $(X,Y,k) = (31, 720, 3)$. For this to work, you have to know that there is no other solution $(31, Y, k)$ which has two different sums with $Y neq 720$, otherwise Beth cannot determine $Y$. In this instance, there is another solution $(1,5,7,18)$ and $(2,3,5,21)$ which has sum $31$, product $630$ $k=4$.
    $endgroup$
    – hexomino
    May 27 at 12:04















2












$begingroup$

Partial Solution



Beth's favorite number is:




Not $1$ or $2$, because then Ana's favorite number is trivial to be $1$. This is because of $1 = 1$ and $2 = 1 + 1$. (In case there can be only $1$ turtle, then $2$ is still possible.)


Thus, Beth's favorite number is $X$ where $X geq 3$. This is because of $X$ can be represented as at least $1 + 1 + dots + 1$ and $1 + (X-1)$ so there are at least $2$ possibilities of Ana's favorite number (which is $1$ or $X-1$).




Because Beth still won't know the ages of stuffed animal turtles, given Ana's favorite number and how many stuffed animal turtles, then we can conclude that:




Let's say there are $k$ turtles. If the turtles' ages are $T = T_1, T_2, dots, T_k$; there should be at least $2$ possibilities of $T$ where both sum and product of them are same.




Also as @hexomino pointed out from comments, Beth then know the answer, means:




For such sum, the product should be unique.




To sums up, we want to find:




$X$, $Y$, and $k$ such that there are at least $2$ possibilities of $T = T_1, T_2, dots, T_k$ that satisfies:

$(1)~X geq 3$
$(2)~T_1 + T_2 + dots + T_k = X$
$(3)~T_1 times T_2 times dots times T_k = Y$
$(4)$ There is no other $Y'$ and $k'$ for such $X$ so that $(2), (3)$ are also satisfied for given $Y'$ and $k'$.







share|improve this answer











$endgroup$








  • 1




    $begingroup$
    I think there is an additional restriction in that once you know $X$, you can determine $Y$ because Beth knows exactly the product at the end. This means that there is an $(X,Y,k)$ with at least two solutions and any other such triplet including $X$ has the same value for $Y$. Also we must have the $k >2$, otherwise Beth could determine the turtle values exactly from knowing $Y$ and $k$.
    $endgroup$
    – hexomino
    May 27 at 11:44










  • $begingroup$
    @hexomino ah, clever! I should try to fix this..
    $endgroup$
    – athin
    May 27 at 11:48










  • $begingroup$
    @hexomino wait no, even if you know $X$, $Y$ still couldn't be determined as indicated by first spoiler..
    $endgroup$
    – athin
    May 27 at 11:52










  • $begingroup$
    so Beth's first question is essentially "If you tell me $Y$ and $k$ can I determine the ages?" If $k<3$, then Ana would say "yes" because it's a determined system.
    $endgroup$
    – hexomino
    May 27 at 11:56






  • 1




    $begingroup$
    As to my first point, take your solution $(X,Y,k) = (31, 720, 3)$. For this to work, you have to know that there is no other solution $(31, Y, k)$ which has two different sums with $Y neq 720$, otherwise Beth cannot determine $Y$. In this instance, there is another solution $(1,5,7,18)$ and $(2,3,5,21)$ which has sum $31$, product $630$ $k=4$.
    $endgroup$
    – hexomino
    May 27 at 12:04













2












2








2





$begingroup$

Partial Solution



Beth's favorite number is:




Not $1$ or $2$, because then Ana's favorite number is trivial to be $1$. This is because of $1 = 1$ and $2 = 1 + 1$. (In case there can be only $1$ turtle, then $2$ is still possible.)


Thus, Beth's favorite number is $X$ where $X geq 3$. This is because of $X$ can be represented as at least $1 + 1 + dots + 1$ and $1 + (X-1)$ so there are at least $2$ possibilities of Ana's favorite number (which is $1$ or $X-1$).




Because Beth still won't know the ages of stuffed animal turtles, given Ana's favorite number and how many stuffed animal turtles, then we can conclude that:




Let's say there are $k$ turtles. If the turtles' ages are $T = T_1, T_2, dots, T_k$; there should be at least $2$ possibilities of $T$ where both sum and product of them are same.




Also as @hexomino pointed out from comments, Beth then know the answer, means:




For such sum, the product should be unique.




To sums up, we want to find:




$X$, $Y$, and $k$ such that there are at least $2$ possibilities of $T = T_1, T_2, dots, T_k$ that satisfies:

$(1)~X geq 3$
$(2)~T_1 + T_2 + dots + T_k = X$
$(3)~T_1 times T_2 times dots times T_k = Y$
$(4)$ There is no other $Y'$ and $k'$ for such $X$ so that $(2), (3)$ are also satisfied for given $Y'$ and $k'$.







share|improve this answer











$endgroup$



Partial Solution



Beth's favorite number is:




Not $1$ or $2$, because then Ana's favorite number is trivial to be $1$. This is because of $1 = 1$ and $2 = 1 + 1$. (In case there can be only $1$ turtle, then $2$ is still possible.)


Thus, Beth's favorite number is $X$ where $X geq 3$. This is because of $X$ can be represented as at least $1 + 1 + dots + 1$ and $1 + (X-1)$ so there are at least $2$ possibilities of Ana's favorite number (which is $1$ or $X-1$).




Because Beth still won't know the ages of stuffed animal turtles, given Ana's favorite number and how many stuffed animal turtles, then we can conclude that:




Let's say there are $k$ turtles. If the turtles' ages are $T = T_1, T_2, dots, T_k$; there should be at least $2$ possibilities of $T$ where both sum and product of them are same.




Also as @hexomino pointed out from comments, Beth then know the answer, means:




For such sum, the product should be unique.




To sums up, we want to find:




$X$, $Y$, and $k$ such that there are at least $2$ possibilities of $T = T_1, T_2, dots, T_k$ that satisfies:

$(1)~X geq 3$
$(2)~T_1 + T_2 + dots + T_k = X$
$(3)~T_1 times T_2 times dots times T_k = Y$
$(4)$ There is no other $Y'$ and $k'$ for such $X$ so that $(2), (3)$ are also satisfied for given $Y'$ and $k'$.








share|improve this answer














share|improve this answer



share|improve this answer








edited May 27 at 12:15

























answered May 27 at 10:52









athinathin

10.9k23793




10.9k23793







  • 1




    $begingroup$
    I think there is an additional restriction in that once you know $X$, you can determine $Y$ because Beth knows exactly the product at the end. This means that there is an $(X,Y,k)$ with at least two solutions and any other such triplet including $X$ has the same value for $Y$. Also we must have the $k >2$, otherwise Beth could determine the turtle values exactly from knowing $Y$ and $k$.
    $endgroup$
    – hexomino
    May 27 at 11:44










  • $begingroup$
    @hexomino ah, clever! I should try to fix this..
    $endgroup$
    – athin
    May 27 at 11:48










  • $begingroup$
    @hexomino wait no, even if you know $X$, $Y$ still couldn't be determined as indicated by first spoiler..
    $endgroup$
    – athin
    May 27 at 11:52










  • $begingroup$
    so Beth's first question is essentially "If you tell me $Y$ and $k$ can I determine the ages?" If $k<3$, then Ana would say "yes" because it's a determined system.
    $endgroup$
    – hexomino
    May 27 at 11:56






  • 1




    $begingroup$
    As to my first point, take your solution $(X,Y,k) = (31, 720, 3)$. For this to work, you have to know that there is no other solution $(31, Y, k)$ which has two different sums with $Y neq 720$, otherwise Beth cannot determine $Y$. In this instance, there is another solution $(1,5,7,18)$ and $(2,3,5,21)$ which has sum $31$, product $630$ $k=4$.
    $endgroup$
    – hexomino
    May 27 at 12:04












  • 1




    $begingroup$
    I think there is an additional restriction in that once you know $X$, you can determine $Y$ because Beth knows exactly the product at the end. This means that there is an $(X,Y,k)$ with at least two solutions and any other such triplet including $X$ has the same value for $Y$. Also we must have the $k >2$, otherwise Beth could determine the turtle values exactly from knowing $Y$ and $k$.
    $endgroup$
    – hexomino
    May 27 at 11:44










  • $begingroup$
    @hexomino ah, clever! I should try to fix this..
    $endgroup$
    – athin
    May 27 at 11:48










  • $begingroup$
    @hexomino wait no, even if you know $X$, $Y$ still couldn't be determined as indicated by first spoiler..
    $endgroup$
    – athin
    May 27 at 11:52










  • $begingroup$
    so Beth's first question is essentially "If you tell me $Y$ and $k$ can I determine the ages?" If $k<3$, then Ana would say "yes" because it's a determined system.
    $endgroup$
    – hexomino
    May 27 at 11:56






  • 1




    $begingroup$
    As to my first point, take your solution $(X,Y,k) = (31, 720, 3)$. For this to work, you have to know that there is no other solution $(31, Y, k)$ which has two different sums with $Y neq 720$, otherwise Beth cannot determine $Y$. In this instance, there is another solution $(1,5,7,18)$ and $(2,3,5,21)$ which has sum $31$, product $630$ $k=4$.
    $endgroup$
    – hexomino
    May 27 at 12:04







1




1




$begingroup$
I think there is an additional restriction in that once you know $X$, you can determine $Y$ because Beth knows exactly the product at the end. This means that there is an $(X,Y,k)$ with at least two solutions and any other such triplet including $X$ has the same value for $Y$. Also we must have the $k >2$, otherwise Beth could determine the turtle values exactly from knowing $Y$ and $k$.
$endgroup$
– hexomino
May 27 at 11:44




$begingroup$
I think there is an additional restriction in that once you know $X$, you can determine $Y$ because Beth knows exactly the product at the end. This means that there is an $(X,Y,k)$ with at least two solutions and any other such triplet including $X$ has the same value for $Y$. Also we must have the $k >2$, otherwise Beth could determine the turtle values exactly from knowing $Y$ and $k$.
$endgroup$
– hexomino
May 27 at 11:44












$begingroup$
@hexomino ah, clever! I should try to fix this..
$endgroup$
– athin
May 27 at 11:48




$begingroup$
@hexomino ah, clever! I should try to fix this..
$endgroup$
– athin
May 27 at 11:48












$begingroup$
@hexomino wait no, even if you know $X$, $Y$ still couldn't be determined as indicated by first spoiler..
$endgroup$
– athin
May 27 at 11:52




$begingroup$
@hexomino wait no, even if you know $X$, $Y$ still couldn't be determined as indicated by first spoiler..
$endgroup$
– athin
May 27 at 11:52












$begingroup$
so Beth's first question is essentially "If you tell me $Y$ and $k$ can I determine the ages?" If $k<3$, then Ana would say "yes" because it's a determined system.
$endgroup$
– hexomino
May 27 at 11:56




$begingroup$
so Beth's first question is essentially "If you tell me $Y$ and $k$ can I determine the ages?" If $k<3$, then Ana would say "yes" because it's a determined system.
$endgroup$
– hexomino
May 27 at 11:56




1




1




$begingroup$
As to my first point, take your solution $(X,Y,k) = (31, 720, 3)$. For this to work, you have to know that there is no other solution $(31, Y, k)$ which has two different sums with $Y neq 720$, otherwise Beth cannot determine $Y$. In this instance, there is another solution $(1,5,7,18)$ and $(2,3,5,21)$ which has sum $31$, product $630$ $k=4$.
$endgroup$
– hexomino
May 27 at 12:04




$begingroup$
As to my first point, take your solution $(X,Y,k) = (31, 720, 3)$. For this to work, you have to know that there is no other solution $(31, Y, k)$ which has two different sums with $Y neq 720$, otherwise Beth cannot determine $Y$. In this instance, there is another solution $(1,5,7,18)$ and $(2,3,5,21)$ which has sum $31$, product $630$ $k=4$.
$endgroup$
– hexomino
May 27 at 12:04











1












$begingroup$


Beth's favourite number can be any number >= 13. The possible age combinations are 6,6,1,[1,1,1,...] and 9,2,2,[1,1,1,...]. You can add as many 1's as you like, because this doesn't change Ana's favourite number. Ana's favourite number must be 36. If it were not, the answer to Beth's question would not give any further information and she would not have asked.




Meanwhile, hexomino gave a good explanation why this is so.






share|improve this answer











$endgroup$












  • $begingroup$
    The problem here is precisely the fact that Beth deduced the number. Consider the four possibilities for $S=13$: 11344 12226 ($P = 48$), and 661 922 ($P = 36$). Since Beth only knows her sum, of which all of these four cases fit, but she also knows that the number of turtles and their product would not uniquely determine the set of agres. This is consistent with both $P=36$ or $P=48$, as both cases have two possible sets of ages. Since there are two possible values for $P$, Beth could not have deduced the ages, contradicting her exclamation that she figured out Ana's number, the product.
    $endgroup$
    – greenturtle3141
    May 27 at 16:24















1












$begingroup$


Beth's favourite number can be any number >= 13. The possible age combinations are 6,6,1,[1,1,1,...] and 9,2,2,[1,1,1,...]. You can add as many 1's as you like, because this doesn't change Ana's favourite number. Ana's favourite number must be 36. If it were not, the answer to Beth's question would not give any further information and she would not have asked.




Meanwhile, hexomino gave a good explanation why this is so.






share|improve this answer











$endgroup$












  • $begingroup$
    The problem here is precisely the fact that Beth deduced the number. Consider the four possibilities for $S=13$: 11344 12226 ($P = 48$), and 661 922 ($P = 36$). Since Beth only knows her sum, of which all of these four cases fit, but she also knows that the number of turtles and their product would not uniquely determine the set of agres. This is consistent with both $P=36$ or $P=48$, as both cases have two possible sets of ages. Since there are two possible values for $P$, Beth could not have deduced the ages, contradicting her exclamation that she figured out Ana's number, the product.
    $endgroup$
    – greenturtle3141
    May 27 at 16:24













1












1








1





$begingroup$


Beth's favourite number can be any number >= 13. The possible age combinations are 6,6,1,[1,1,1,...] and 9,2,2,[1,1,1,...]. You can add as many 1's as you like, because this doesn't change Ana's favourite number. Ana's favourite number must be 36. If it were not, the answer to Beth's question would not give any further information and she would not have asked.




Meanwhile, hexomino gave a good explanation why this is so.






share|improve this answer











$endgroup$




Beth's favourite number can be any number >= 13. The possible age combinations are 6,6,1,[1,1,1,...] and 9,2,2,[1,1,1,...]. You can add as many 1's as you like, because this doesn't change Ana's favourite number. Ana's favourite number must be 36. If it were not, the answer to Beth's question would not give any further information and she would not have asked.




Meanwhile, hexomino gave a good explanation why this is so.







share|improve this answer














share|improve this answer



share|improve this answer








edited May 27 at 14:12

























answered May 27 at 11:27









GerhardGerhard

30918




30918











  • $begingroup$
    The problem here is precisely the fact that Beth deduced the number. Consider the four possibilities for $S=13$: 11344 12226 ($P = 48$), and 661 922 ($P = 36$). Since Beth only knows her sum, of which all of these four cases fit, but she also knows that the number of turtles and their product would not uniquely determine the set of agres. This is consistent with both $P=36$ or $P=48$, as both cases have two possible sets of ages. Since there are two possible values for $P$, Beth could not have deduced the ages, contradicting her exclamation that she figured out Ana's number, the product.
    $endgroup$
    – greenturtle3141
    May 27 at 16:24
















  • $begingroup$
    The problem here is precisely the fact that Beth deduced the number. Consider the four possibilities for $S=13$: 11344 12226 ($P = 48$), and 661 922 ($P = 36$). Since Beth only knows her sum, of which all of these four cases fit, but she also knows that the number of turtles and their product would not uniquely determine the set of agres. This is consistent with both $P=36$ or $P=48$, as both cases have two possible sets of ages. Since there are two possible values for $P$, Beth could not have deduced the ages, contradicting her exclamation that she figured out Ana's number, the product.
    $endgroup$
    – greenturtle3141
    May 27 at 16:24















$begingroup$
The problem here is precisely the fact that Beth deduced the number. Consider the four possibilities for $S=13$: 11344 12226 ($P = 48$), and 661 922 ($P = 36$). Since Beth only knows her sum, of which all of these four cases fit, but she also knows that the number of turtles and their product would not uniquely determine the set of agres. This is consistent with both $P=36$ or $P=48$, as both cases have two possible sets of ages. Since there are two possible values for $P$, Beth could not have deduced the ages, contradicting her exclamation that she figured out Ana's number, the product.
$endgroup$
– greenturtle3141
May 27 at 16:24




$begingroup$
The problem here is precisely the fact that Beth deduced the number. Consider the four possibilities for $S=13$: 11344 12226 ($P = 48$), and 661 922 ($P = 36$). Since Beth only knows her sum, of which all of these four cases fit, but she also knows that the number of turtles and their product would not uniquely determine the set of agres. This is consistent with both $P=36$ or $P=48$, as both cases have two possible sets of ages. Since there are two possible values for $P$, Beth could not have deduced the ages, contradicting her exclamation that she figured out Ana's number, the product.
$endgroup$
– greenturtle3141
May 27 at 16:24

















draft saved

draft discarded
















































Thanks for contributing an answer to Puzzling Stack Exchange!


  • Please be sure to answer the question. Provide details and share your research!

But avoid


  • Asking for help, clarification, or responding to other answers.

  • Making statements based on opinion; back them up with references or personal experience.

Use MathJax to format equations. MathJax reference.


To learn more, see our tips on writing great answers.




draft saved


draft discarded














StackExchange.ready(
function ()
StackExchange.openid.initPostLogin('.new-post-login', 'https%3a%2f%2fpuzzling.stackexchange.com%2fquestions%2f84401%2fa-mathematical-discussion-fill-in-the-blank%23new-answer', 'question_page');

);

Post as a guest















Required, but never shown





















































Required, but never shown














Required, but never shown












Required, but never shown







Required, but never shown

































Required, but never shown














Required, but never shown












Required, but never shown







Required, but never shown







Popular posts from this blog

Get product attribute by attribute group code in magento 2get product attribute by product attribute group in magento 2Magento 2 Log Bundle Product Data in List Page?How to get all product attribute of a attribute group of Default attribute set?Magento 2.1 Create a filter in the product grid by new attributeMagento 2 : Get Product Attribute values By GroupMagento 2 How to get all existing values for one attributeMagento 2 get custom attribute of a single product inside a pluginMagento 2.3 How to get all the Multi Source Inventory (MSI) locations collection in custom module?Magento2: how to develop rest API to get new productsGet product attribute by attribute group code ( [attribute_group_code] ) in magento 2

Category:9 (number) SubcategoriesMedia in category "9 (number)"Navigation menuUpload mediaGND ID: 4485639-8Library of Congress authority ID: sh85091979ReasonatorScholiaStatistics

Magento 2.3: How do i solve this, Not registered handle, on custom form?How can i rewrite TierPrice Block in Magento2magento 2 captcha not rendering if I override layout xmlmain.CRITICAL: Plugin class doesn't existMagento 2 : Problem while adding custom button order view page?Magento 2.2.5: Overriding Admin Controller sales/orderMagento 2.2.5: Add, Update and Delete existing products Custom OptionsMagento 2.3 : File Upload issue in UI Component FormMagento2 Not registered handleHow to configured Form Builder Js in my custom magento 2.3.0 module?Magento 2.3. How to create image upload field in an admin form